Đến nội dung

Hình ảnh

Inequalities From 2016 Mathematical Olympiads

* * * * * 3 Bình chọn

  • Please log in to reply
Chủ đề này có 139 trả lời

#81
Nguyenhuyen_AG

Nguyenhuyen_AG

    Trung úy

  • Thành viên nổi bật 2016
  • 945 Bài viết

Bài 37 (Turkmenistan RMO). Nếu $a,b,c$ là độ dài ba cạnh của tam giác thì

\[\sqrt{\frac{a}{b+c-a}}+\sqrt{\frac{b}{c+a-b}}+\sqrt{\frac{c}{a+b-c}} \geqslant  3.\]
 


Nguyen Van Huyen
Ho Chi Minh City University Of Transport

#82
PlanBbyFESN

PlanBbyFESN

    Thiếu úy

  • Điều hành viên OLYMPIC
  • 637 Bài viết

Bài 37 (Turkmenistan RMO). Nếu $a,b,c$ là độ dài ba cạnh của tam giác thì

\[\sqrt{\frac{a}{b+c-a}}+\sqrt{\frac{b}{c+a-b}}+\sqrt{\frac{c}{a+b-c}} \geqslant  3.\]
 

 

Bài này rất quen thuộc:

 

$\sqrt{\frac{a}{b+c-a}}+\sqrt{\frac{b}{c+a-b}}+\sqrt{\frac{c}{a+b-c}}\geq 3\sqrt[6]{\frac{abc}{(a+b-c)(b+c-a)(c+a-b)}}$

 

$\left\{\begin{matrix} (b+c-a)(a+b-c)\leq b^{2} & & \\ (b+c-a)(a+c-b)\leq c^{2} & & \\ (c+a-b)(a+b-c)\leq a^{2} & & \end{matrix}\right.$

 

$\Rightarrow abc\geq (a+b-c)(b+c-a)(c+a-b)$ 

 

$\Rightarrow \sqrt{\frac{a}{b+c-a}}+\sqrt{\frac{b}{c+a-b}}+\sqrt{\frac{c}{a+b-c}} \geqslant 3\blacksquare$


Bài viết đã được chỉnh sửa nội dung bởi PlanBbyFESN: 27-05-2016 - 23:45

:huh:


#83
Nguyenhuyen_AG

Nguyenhuyen_AG

    Trung úy

  • Thành viên nổi bật 2016
  • 945 Bài viết

Bài 38 (Turkey EGMO TST). Chứng minh rằng
\[x^4y+y^4z+z^4x+xyz(x^3+y^3+z^3) \geqslant (x+y+z)(3xyz-1),\]
 với mọi số thực dương $x, y, z$ thay đổi bất kỳ.


Nguyen Van Huyen
Ho Chi Minh City University Of Transport

#84
minhrongcon2000

minhrongcon2000

    Thượng sĩ

  • Thành viên
  • 213 Bài viết

Bài 38 (Turkey EGMO TST). Chứng minh rằng
\[x^4y+y^4z+z^4x+xyz(x^3+y^3+z^3) \geqslant (x+y+z)(3xyz-1),\]
 với mọi số thực dương $x, y, z$ thay đổi bất kỳ.

$x^{4}y+y^{4}z+z^{4}x+xyz(x^{3}+y^{3}+z^{3})+(x+y+z)$

$\geqslant x^{4}y+y^{4}z+z^{4}x+xyz(xy^{2}+yz^{2}+zx^{2})+(x+y+z)$

$\geqslant (x^{4}y+xy^{2}z^{3}+x)+(y^{4}z+yz^{2}x^{3}+y)+(z^{4}x+zx^{2}y^{3}+z)$

$\geqslant 3xyz(x+y+z)$

$\Rightarrow x^4y+y^4z+z^4x+xyz(x^3+y^3+z^3) \geqslant (x+y+z)(3xyz-1)$


$\lim_{x \to \infty } Love =+\infty$


#85
Nguyenhuyen_AG

Nguyenhuyen_AG

    Trung úy

  • Thành viên nổi bật 2016
  • 945 Bài viết

Bài 39 (Mediterranean MO). Cho ba số thực dương $a,b,c$ thỏa mãn $a+b+c=3.$ Chứng minh rằng
\[\sqrt{\frac{b}{a^2+3}}+\sqrt{\frac{c}{b^2+3}}+\sqrt{\frac{a}{c^2+3}} \leqslant \frac32\sqrt[4]{\frac{1}{abc}}.\]

 


Nguyen Van Huyen
Ho Chi Minh City University Of Transport

#86
Ankh

Ankh

    Hạ sĩ

  • Thành viên
  • 85 Bài viết

Bài 39 (Mediterranean MO). Cho ba số thực dương $a,b,c$ thỏa mãn $a+b+c=3.$ Chứng minh rằng
\[\sqrt{\frac{b}{a^2+3}}+\sqrt{\frac{c}{b^2+3}}+\sqrt{\frac{a}{c^2+3}} \leqslant \frac32\sqrt[4]{\frac{1}{abc}}.\]

 

 Vế phải của bất đẳng thức thực ra là $\dfrac{1}{2}.\dfrac{3}{\sqrt[4]{abc}}=\dfrac{a+b+c}{2\sqrt[4]{abc}}$

 Áp dụng bất đẳng thức AM-GM thì $a^2+3\geq 4\sqrt{a}$ nên $VT\leq \sum \dfrac{\sqrt{b}}{2\sqrt[4]{a}}$

 Đặt $\sqrt[4]{a}=x,\cdots $ thì ta chỉ cần chứng minh $\sum \dfrac{y^2}{x}\leq \dfrac{x^4+y^4+z^4}{xyz}\Leftrightarrow x^3y+y^3z+z^3x\leq x^4+y^4+z^4$

 Bất đẳng thức trên chứng minh khá dễ bằng AM-GM :)



#87
Nguyenngoctu

Nguyenngoctu

    Trung sĩ

  • Thành viên
  • 118 Bài viết

Bài 39 (Mediterranean MO). Cho ba số thực dương $a,b,c$ thỏa mãn $a+b+c=3.$ Chứng minh rằng
\[\sqrt{\frac{b}{a^2+3}}+\sqrt{\frac{c}{b^2+3}}+\sqrt{\frac{a}{c^2+3}} \leqslant \frac32\sqrt[4]{\frac{1}{abc}}.\]

 

Đặt $$S = \sqrt {\frac{b}{{{a^2} + 3}}} + \sqrt {\frac{c}{{{b^2} + 3}}} + \sqrt {\frac{a}{{{c^2} + 3}}} $$. Từ a+b+c=3 ta có $$9 = {\left( {a + b + c} \right)^2} \ge 3\left( {ab + bc + ca} \right) \Rightarrow ab + bc + ca \le 3 = a + b + c$$. Và $${a^2} + 3 = {a^2} + 1 + 1 + 1 \ge 4\sqrt a $$. Do đó:

$$\begin{array}{l} S \le \frac{1}{2}\sum {\sqrt {\frac{b}{{\sqrt a }}} } \Rightarrow {S^2} \le \frac{1}{4}{\left( {\sum {\sqrt {\frac{b}{{\sqrt a }}} } } \right)^2} \le \frac{1}{4}\left( {a + b + c} \right)\left( {\frac{1}{{\sqrt a }} + \frac{1}{{\sqrt b }}\frac{1}{{\sqrt c }}} \right) = \frac{3}{4}\left( {\frac{1}{{\sqrt a }} + \frac{1}{{\sqrt b }}\frac{1}{{\sqrt c }}} \right)\\ \Rightarrow {S^4} \le \frac{9}{{16}}{\left( {\frac{1}{{\sqrt a }} + \frac{1}{{\sqrt b }}\frac{1}{{\sqrt c }}} \right)^2} \le \frac{{27}}{{16}}\left( {\frac{1}{a} + \frac{1}{b} + \frac{1}{c}} \right) = \frac{{27}}{{16}}.\frac{{ab + bc + ca}}{{abc}} \le \frac{{81}}{{16}}.\frac{1}{{abc}} \end{array}$$.

Vậy $$S \le \frac{3}{2}\sqrt[4]{{\frac{1}{{abc}}}}$$. Bài toán được chứng minh.



#88
Nguyenngoctu

Nguyenngoctu

    Trung sĩ

  • Thành viên
  • 118 Bài viết

Bài 37 (Turkmenistan RMO). Nếu $a,b,c$ là độ dài ba cạnh của tam giác thì

\[\sqrt{\frac{a}{b+c-a}}+\sqrt{\frac{b}{c+a-b}}+\sqrt{\frac{c}{a+b-c}} \geqslant  3.\]
 

Ta có $\[\begin{array}{l} \frac{{b + c - a}}{a} + 1 = \frac{{b + c}}{a} \ge 2\sqrt {\frac{{b + c - a}}{a}} \\ \Rightarrow \sqrt {\frac{a}{{b + c - a}}} \ge \frac{{2a}}{{b + c}} \Rightarrow \sqrt {\frac{a}{{b + c - a}}} + \sqrt {\frac{b}{{c + a - b}}} + \sqrt {\frac{c}{{a + b - c}}} \ge 2\sum {\frac{a}{{b + c}}} \ge 3 \end{array}\]$. Bài toán được chứng minh.



#89
Nguyenhuyen_AG

Nguyenhuyen_AG

    Trung úy

  • Thành viên nổi bật 2016
  • 945 Bài viết

Bài 40 (Syrian MO). Cho ba số thực dương $a,\,b,\,c$ thỏa mãn điều kiện $a+b+c=3.$ Chứng minh rằng
\[\frac{a^3+b^3}{a^2+ab+b^2}+\frac{b^3+c^3}{b^2+bc+c^2}+\frac{c^3+a^3}{c^2+ca+a^2} \geqslant 2.\]
Đẳng thức xảy ra khi nào ?
 


Nguyen Van Huyen
Ho Chi Minh City University Of Transport

#90
tpdtthltvp

tpdtthltvp

    Trung úy

  • Điều hành viên THCS
  • 831 Bài viết

Bài 40 (Syrian MO). Cho ba số thực dương $a,\,b,\,c$ thỏa mãn điều kiện $a+b+c=3.$ Chứng minh rằng
\[\frac{a^3+b^3}{a^2+ab+b^2}+\frac{b^3+c^3}{b^2+bc+c^2}+\frac{c^3+a^3}{c^2+ca+a^2} \geqslant 2.\]
Đẳng thức xảy ra khi nào ?
 

Bài nãy cũng quen thuộc :) :

Ta có:

$$\frac{a^2-ab+b^2}{a^2+ab+b^2}\geq \frac{1}{3}\Leftrightarrow \frac{2(a-b)^2}{3(a^2+ab+b^2)}\geq 0,\text{đúng}$$

Do đó:

$$\sum \frac{a^3+b^3}{a^2+ab+b^2}=\sum (a+b).\frac{a^2-ab+b^2}{a^2+ab+b^2}\geq \sum \frac{1}{3}(a+b)=2$$

Đẳng thức xảy ra khi $a=b=c=1$


Bài viết đã được chỉnh sửa nội dung bởi tpdtthltvp: 11-06-2016 - 07:53

$\color{red}{\mathrm{\text{How I wish I could recollect, of circle roud}}}$

$\color{red}{\mathrm{\text{The exact relation Archimede unwound ! }}}$

 


#91
Nguyenhuyen_AG

Nguyenhuyen_AG

    Trung úy

  • Thành viên nổi bật 2016
  • 945 Bài viết

Bài 41 (PAMO). Cho ba số dương $x,y,z$ thỏa mãn điều kiện $xyz=1.$ Chứng minh rằng
$$\frac{1}{(x+1)^2+y^2+1} + \frac{1}{(y+1)^2+z^2+1} + \frac{1}{(z+1)^2+x^2+1} \leqslant {\frac{1}{2}}.$$
Bài 42 (EMMO). Cho $n$ số thực dương  $a_1,\,a_2,\ldots,\,a_n.$ Chứng minh rằng $$\prod_{i=1}^{n} \left(1+\frac{1}{a_i}\right)^{a_{i+1}-a_i} \geqslant 1,$$
trong đó $a_{n+1}=a_1.$


Bài viết đã được chỉnh sửa nội dung bởi Nguyenhuyen_AG: 13-06-2016 - 01:25

Nguyen Van Huyen
Ho Chi Minh City University Of Transport

#92
tpdtthltvp

tpdtthltvp

    Trung úy

  • Điều hành viên THCS
  • 831 Bài viết

Bài 41 (PAMO). Cho ba số dương $x,y,z$ thỏa mãn điều kiện $xyz=1.$ Chứng minh rằng
$$\frac{1}{(x+1)^2+y^2+1} + \frac{1}{(y+1)^2+z^2+1} + \frac{1}{(z+1)^2+x^2+1} \leqslant {\frac{1}{2}}.$$
 

Bài 41 (PAMO):  Áp dụng $AM-GM:$

$$\sum \frac{1}{(x+1)^2+y^2+1}=\sum \frac{1}{x^2+2x+1+y^2+1}\leq \frac{1}{2}\sum \frac{1}{xy+x+1}=\frac{1}{2}(\text{do }xyz=1)$$

Đẳng thức xảy ra khi $x=y=z=1.$

 

Bài 42 (EMMO). Cho $n$ số thực dương  $a_1,\,a_2,\ldots,\,a_n.$ Chứng minh rằng $$\prod_{i=1}^{n} \left(1+\frac{1}{a_i}\right)^{a_{i+1}-a_i} \geqslant 1,$$
trong đó $a_{n+1}=a_1.$

Bài 42 (EMMO): 

Áp dụng BĐT $Bernoulli,$ ta có:

$$\left(1+\frac{1}{a_i}\right)^{a_{i+1}-a_i}\geq 1+\frac{a_{i+1}-a_i}{a_i}=\frac{a_{i+1}}{a_i}$$

Do đó:

$$\prod_{i=1}^{n} \left(1+\frac{1}{a_i}\right)^{a_{i+1}-a_i}\geq \frac{a_2}{a_1}.\frac{a_3}{a_2}\cdots \frac{a_1}{a_n}=1$$


Bài viết đã được chỉnh sửa nội dung bởi tpdtthltvp: 13-06-2016 - 06:26

$\color{red}{\mathrm{\text{How I wish I could recollect, of circle roud}}}$

$\color{red}{\mathrm{\text{The exact relation Archimede unwound ! }}}$

 


#93
Nguyenhuyen_AG

Nguyenhuyen_AG

    Trung úy

  • Thành viên nổi bật 2016
  • 945 Bài viết

Bài 43 (Romanian JBMO TST). Với $a,\,b,\,c$ là ba số thực dương thỏa mãn điều kiện $abc \geqslant 1.$ Chứng minh rằng
$$\dfrac{1}{a^3+2b^3+6}+\dfrac{1}{b^3+2c^3+6}+\dfrac{1}{c^3+2a^3+6} \le \dfrac{1}{3}.$$
 


Nguyen Van Huyen
Ho Chi Minh City University Of Transport

#94
fatcat12345

fatcat12345

    Binh nhất

  • Banned
  • 46 Bài viết

Bài 43 (Romanian JBMO TST). Với $a,\,b,\,c$ là ba số thực dương thỏa mãn điều kiện $abc \geqslant 1.$ Chứng minh rằng
$$\dfrac{1}{a^3+2b^3+6}+\dfrac{1}{b^3+2c^3+6}+\dfrac{1}{c^3+2a^3+6} \le \dfrac{1}{3}.$$
 

Vì $abc \geq 1$ nên tồn tại $k \geq 1$ và $a',b',c'>0$ sao cho $a=ka', b=kb', c=kc'$ và $a'b'c'=1$.

Suy ra $a\geq a', b\geq b', c\geq c'$.

Do đó

\[\sum_{a,b,c}\frac{1}{a^3+2b^3+6}\leq \sum_{a',b',c'}\frac{1}{a'^3+2b'^3+6}=\sum_{a',b',c'}\frac{1}{(a'^3+b'^3+1)+(b'^3+1+1)+3}\leq \frac{1}{3}\sum_{a',b',c'}\frac{1}{a'b'+b'+1}=\frac{1}{3}.\]
Ta có đpcm.

Bài viết đã được chỉnh sửa nội dung bởi fatcat12345: 15-06-2016 - 14:19


#95
Nguyenngoctu

Nguyenngoctu

    Trung sĩ

  • Thành viên
  • 118 Bài viết

Bài 40 (Syrian MO). Cho ba số thực dương $a,\,b,\,c$ thỏa mãn điều kiện $a+b+c=3.$ Chứng minh rằng
\[\frac{a^3+b^3}{a^2+ab+b^2}+\frac{b^3+c^3}{b^2+bc+c^2}+\frac{c^3+a^3}{c^2+ca+a^2} \geqslant 2.\]
Đẳng thức xảy ra khi nào ?
 

$$\begin{array}{l} \sum {\frac{{{a^3} + {b^3}}}{{{a^2} + ab + {b^2}}}} = \sum {\frac{{\left( {a + b} \right)\left( {{a^2} + ab + {b^2}} \right) - 2ab\left( {a + b} \right)}}{{{a^2} + ab + {b^2}}}} = 2\sum a - \sum {\frac{{2ab\left( {a + b} \right)}}{{{a^2} + ab + {b^2}}}} \\ \ge 2\sum a - \sum {\frac{{2ab\left( {a + b} \right)}}{{3ab}}} = 2\sum a - \frac{4}{3}\sum a = \frac{2}{3}\sum a = 2 \end{array}$$


Bài viết đã được chỉnh sửa nội dung bởi Nguyenngoctu: 19-06-2016 - 12:35


#96
Nguyenngoctu

Nguyenngoctu

    Trung sĩ

  • Thành viên
  • 118 Bài viết

Bài 43 (Romanian JBMO TST). Với $a,\,b,\,c$ là ba số thực dương thỏa mãn điều kiện $abc \geqslant 1.$ Chứng minh rằng
$$\dfrac{1}{a^3+2b^3+6}+\dfrac{1}{b^3+2c^3+6}+\dfrac{1}{c^3+2a^3+6} \le \dfrac{1}{3}.$$
 

Ta có $$\sum {\frac{1}{{{a^3} + 2{b^3} + 6}}} \le \frac{1}{3} \Leftrightarrow \sum {\frac{{{a^3} + 2{b^3}}}{{{a^3} + 2{b^3} + 6}}} \ge 1$$

Ta chỉ cần chứng minh $$\sum {\frac{{{a^2} + 2{b^2}}}{{{a^2} + 2{b^2} + 6}}} \ge 1$$ là xong!

Ta có $$\sum {\frac{{{a^2}}}{{{a^2} + 2{b^2} + 6}}} \ge \frac{{{{\left( {\sum a } \right)}^2}}}{{3\sum {{a^2}} + 18}} = \frac{{\sum {{a^2}} + 2\sum {ab} }}{{3\sum {{a^2}} + 18}} \ge \frac{1}{3}$$ đúng do $$\sum {ab} \ge 3$$.

Tương tự $$\sum {\frac{{2{b^2}}}{{{a^2} + 2{b^2} + 6}}} \ge \frac{2}{3}$$. Cộng vế với vế ta có điều phải chứng minh. 



#97
Nguyenngoctu

Nguyenngoctu

    Trung sĩ

  • Thành viên
  • 118 Bài viết

$$\frac{1}{{{a^k} + 2{b^k} + 6}} + \frac{1}{{{b^k} + 2{c^k} + 6}} + \frac{1}{{{c^k} + 2{a^k} + 6}} \le \frac{1}{3}$$  :luoi: ?????



#98
Nguyenhuyen_AG

Nguyenhuyen_AG

    Trung úy

  • Thành viên nổi bật 2016
  • 945 Bài viết

Bài 44 (BMOJ TST). Cho ba số thực $a \geqslant b \geqslant 1 \geqslant c\geqslant 0$ thỏa mãn điều kiện $a+b+c=3.$

  1. Chứng minh rằng $2 \leqslant ab + bc +ca \leqslant 3.$
     
  2. Chứng minh rằng \[\frac{24}{a^3+b^3+c^3} + \frac{25}{ab+bc+ca} \geqslant 14.\] Đẳng thức xảy ra khi nào ?

Bài viết đã được chỉnh sửa nội dung bởi Nguyenhuyen_AG: 23-06-2016 - 15:49

Nguyen Van Huyen
Ho Chi Minh City University Of Transport

#99
fatcat12345

fatcat12345

    Binh nhất

  • Banned
  • 46 Bài viết

 

Bài 44 (BMOJ TST). Cho ba số thực $a \geqslant b \geqslant 1 \geqslant c\geqslant 0$ thỏa mãn điều kiện $a+b+c=3.$

  1. Chứng minh rằng $2 \leqslant ab + bc +ca \leqslant 3.$
     
  2. Chứng minh rằng \[\frac{24}{a^3+b^3+c^3} + \frac{25}{ab+bc+ca} \geqslant 14.\] Đẳng thức xảy ra khi nào ?

 

Đặt $q=ab+bc+ca, r=abc.$

$1.$ Vì $(1-a)(1-b)(1-c)=q-r-2\geq 0$ nên $q\geq r+2\geq 2.$

Lại có $3(ab+bc+ca)\leq (a+b+c)^2$ nên $ab+bc+ca\leq 3.$

$2.$ Bất đẳng thức tương đương 

$$\frac{8}{9-3q+r}+\frac{25}{q}\geq 14$$

mà ta lại có $r\leq q-2$ nên ta chỉ cần chứng minh

$$\frac{8}{7-2q}+\frac{25}{q}\geq 14$$

$$\Leftrightarrow 7(2q-5)^2\geq 0.$$

Đẳng thức xảy ra khi và chỉ khi $a=\frac{2+\sqrt{2}}{2}, b=1, c=\frac{2-\sqrt{2}}{2}.$


Bài viết đã được chỉnh sửa nội dung bởi fatcat12345: 23-06-2016 - 16:26


#100
Nguyenhuyen_AG

Nguyenhuyen_AG

    Trung úy

  • Thành viên nổi bật 2016
  • 945 Bài viết

Bài 45 (Turkey NMO First Stage). Cho ba số thực dương $a,b,c$ thỏa mãn điều kiện $abc=2.$ Tìm giá trị nhỏ nhất của biểu thức $$a^2+2b^2+4c^2-6b.$$
 


Nguyen Van Huyen
Ho Chi Minh City University Of Transport




0 người đang xem chủ đề

0 thành viên, 0 khách, 0 thành viên ẩn danh